Tricky analysis proof involving the sum of integrals












1












$begingroup$


The question is as follows:




Let $P:a=t_{0}<t_{1}<...<t_{m}=b $ be a partition of $[a,b]$, $f_{j}$ be a Riemann integrable function on $[t_{j-1},t_{j}] forall j=1,...,m$, and let $f:[a,b]rightarrow mathbb{R}$ be a function with the property
$$frestriction_{(t_{j-1},t_{j})}=f_jrestriction_{(t_{j-1},t_{j})} forall j=1,...m.$$
Prove that
$$int_{a}^{b}f(x) dx=sum_{j=1}^{m}int_{t_{j-1}}^{t_{j}}f_{j}(x)dx.$$




I already know that $f$ is Riemann integrable iff $frestriction_{(t_{j-1},t_{j})}$ is Riemann integrable for each $j=1,...m.$ I have also proved this, so I believe the proof just needs to be shown for the case $m=1.$



Does anyone know how to tackle this?



Thanks in advance!










share|cite|improve this question









$endgroup$












  • $begingroup$
    There is absolutely nothing to prove when $m=1$ because $f_1=f$ in that case.
    $endgroup$
    – Kavi Rama Murthy
    Dec 9 '18 at 0:07










  • $begingroup$
    @KaviRamaMurthy What is the correct approach then?
    $endgroup$
    – MathIsLife12
    Dec 9 '18 at 1:35
















1












$begingroup$


The question is as follows:




Let $P:a=t_{0}<t_{1}<...<t_{m}=b $ be a partition of $[a,b]$, $f_{j}$ be a Riemann integrable function on $[t_{j-1},t_{j}] forall j=1,...,m$, and let $f:[a,b]rightarrow mathbb{R}$ be a function with the property
$$frestriction_{(t_{j-1},t_{j})}=f_jrestriction_{(t_{j-1},t_{j})} forall j=1,...m.$$
Prove that
$$int_{a}^{b}f(x) dx=sum_{j=1}^{m}int_{t_{j-1}}^{t_{j}}f_{j}(x)dx.$$




I already know that $f$ is Riemann integrable iff $frestriction_{(t_{j-1},t_{j})}$ is Riemann integrable for each $j=1,...m.$ I have also proved this, so I believe the proof just needs to be shown for the case $m=1.$



Does anyone know how to tackle this?



Thanks in advance!










share|cite|improve this question









$endgroup$












  • $begingroup$
    There is absolutely nothing to prove when $m=1$ because $f_1=f$ in that case.
    $endgroup$
    – Kavi Rama Murthy
    Dec 9 '18 at 0:07










  • $begingroup$
    @KaviRamaMurthy What is the correct approach then?
    $endgroup$
    – MathIsLife12
    Dec 9 '18 at 1:35














1












1








1





$begingroup$


The question is as follows:




Let $P:a=t_{0}<t_{1}<...<t_{m}=b $ be a partition of $[a,b]$, $f_{j}$ be a Riemann integrable function on $[t_{j-1},t_{j}] forall j=1,...,m$, and let $f:[a,b]rightarrow mathbb{R}$ be a function with the property
$$frestriction_{(t_{j-1},t_{j})}=f_jrestriction_{(t_{j-1},t_{j})} forall j=1,...m.$$
Prove that
$$int_{a}^{b}f(x) dx=sum_{j=1}^{m}int_{t_{j-1}}^{t_{j}}f_{j}(x)dx.$$




I already know that $f$ is Riemann integrable iff $frestriction_{(t_{j-1},t_{j})}$ is Riemann integrable for each $j=1,...m.$ I have also proved this, so I believe the proof just needs to be shown for the case $m=1.$



Does anyone know how to tackle this?



Thanks in advance!










share|cite|improve this question









$endgroup$




The question is as follows:




Let $P:a=t_{0}<t_{1}<...<t_{m}=b $ be a partition of $[a,b]$, $f_{j}$ be a Riemann integrable function on $[t_{j-1},t_{j}] forall j=1,...,m$, and let $f:[a,b]rightarrow mathbb{R}$ be a function with the property
$$frestriction_{(t_{j-1},t_{j})}=f_jrestriction_{(t_{j-1},t_{j})} forall j=1,...m.$$
Prove that
$$int_{a}^{b}f(x) dx=sum_{j=1}^{m}int_{t_{j-1}}^{t_{j}}f_{j}(x)dx.$$




I already know that $f$ is Riemann integrable iff $frestriction_{(t_{j-1},t_{j})}$ is Riemann integrable for each $j=1,...m.$ I have also proved this, so I believe the proof just needs to be shown for the case $m=1.$



Does anyone know how to tackle this?



Thanks in advance!







real-analysis integration riemann-integration






share|cite|improve this question













share|cite|improve this question











share|cite|improve this question




share|cite|improve this question










asked Dec 8 '18 at 19:59









MathIsLife12MathIsLife12

613211




613211












  • $begingroup$
    There is absolutely nothing to prove when $m=1$ because $f_1=f$ in that case.
    $endgroup$
    – Kavi Rama Murthy
    Dec 9 '18 at 0:07










  • $begingroup$
    @KaviRamaMurthy What is the correct approach then?
    $endgroup$
    – MathIsLife12
    Dec 9 '18 at 1:35


















  • $begingroup$
    There is absolutely nothing to prove when $m=1$ because $f_1=f$ in that case.
    $endgroup$
    – Kavi Rama Murthy
    Dec 9 '18 at 0:07










  • $begingroup$
    @KaviRamaMurthy What is the correct approach then?
    $endgroup$
    – MathIsLife12
    Dec 9 '18 at 1:35
















$begingroup$
There is absolutely nothing to prove when $m=1$ because $f_1=f$ in that case.
$endgroup$
– Kavi Rama Murthy
Dec 9 '18 at 0:07




$begingroup$
There is absolutely nothing to prove when $m=1$ because $f_1=f$ in that case.
$endgroup$
– Kavi Rama Murthy
Dec 9 '18 at 0:07












$begingroup$
@KaviRamaMurthy What is the correct approach then?
$endgroup$
– MathIsLife12
Dec 9 '18 at 1:35




$begingroup$
@KaviRamaMurthy What is the correct approach then?
$endgroup$
– MathIsLife12
Dec 9 '18 at 1:35










1 Answer
1






active

oldest

votes


















1












$begingroup$

This is just a generalization of integrability of $f$ on $[a,c]$ and $[c,b]$ implies integrability on $[a,b]$ and
$$int_a^b f(x) , dx = int_a^c f(x) , dx + int_c^b f(x) , dx $$



There exist partitions $P_j$ of $[t_{j-1},t_j]$ such that for $j=1,ldots,m$, upper and lower Darboux sums satisfy



$$U(P_j,f_j) - L(P_j,f_j) < frac{epsilon}{m}$$



With the partition $P = bigcup_{j=1}^n P_j$ we have



$$U(P,f) - L(P,f) = sum_{j=1}^n [ U(P_j,f_j) - L(P_j,f_j)] < epsilon$$



and $f$ is integrable by the Riemann criterion.



We must also have



$$L(P,f) leqslant int_a^b f(x) , dx leqslant U(P,f), \ L(P,f) = sum_{j=1}^n L(P_j,f_j) = sum_{j=1}^n int_{t_{j-1}}^{t_j} f_j(x) , dx leqslant sum_{j=1}^n U(P_j,f_j) = U(P,f)$$



Thus,



$$-epsilon < -[U(P,f) - L(P,f)] < int_a^b f(x) , dx - sum_{j=1}^n int_{t_{j-1}}^{t_j} f_j(x) , dx < U(P,f) - L(P,f) < epsilon,$$



which implies (since $epsilon$ can be chosen arbitrarily close to $0$)



$$int_a^b f(x) , dx = sum_{j=1}^n int_{t_{j-1}}^{t_j} f_j(x) , dx $$






share|cite|improve this answer









$endgroup$













    Your Answer





    StackExchange.ifUsing("editor", function () {
    return StackExchange.using("mathjaxEditing", function () {
    StackExchange.MarkdownEditor.creationCallbacks.add(function (editor, postfix) {
    StackExchange.mathjaxEditing.prepareWmdForMathJax(editor, postfix, [["$", "$"], ["\\(","\\)"]]);
    });
    });
    }, "mathjax-editing");

    StackExchange.ready(function() {
    var channelOptions = {
    tags: "".split(" "),
    id: "69"
    };
    initTagRenderer("".split(" "), "".split(" "), channelOptions);

    StackExchange.using("externalEditor", function() {
    // Have to fire editor after snippets, if snippets enabled
    if (StackExchange.settings.snippets.snippetsEnabled) {
    StackExchange.using("snippets", function() {
    createEditor();
    });
    }
    else {
    createEditor();
    }
    });

    function createEditor() {
    StackExchange.prepareEditor({
    heartbeatType: 'answer',
    autoActivateHeartbeat: false,
    convertImagesToLinks: true,
    noModals: true,
    showLowRepImageUploadWarning: true,
    reputationToPostImages: 10,
    bindNavPrevention: true,
    postfix: "",
    imageUploader: {
    brandingHtml: "Powered by u003ca class="icon-imgur-white" href="https://imgur.com/"u003eu003c/au003e",
    contentPolicyHtml: "User contributions licensed under u003ca href="https://creativecommons.org/licenses/by-sa/3.0/"u003ecc by-sa 3.0 with attribution requiredu003c/au003e u003ca href="https://stackoverflow.com/legal/content-policy"u003e(content policy)u003c/au003e",
    allowUrls: true
    },
    noCode: true, onDemand: true,
    discardSelector: ".discard-answer"
    ,immediatelyShowMarkdownHelp:true
    });


    }
    });














    draft saved

    draft discarded


















    StackExchange.ready(
    function () {
    StackExchange.openid.initPostLogin('.new-post-login', 'https%3a%2f%2fmath.stackexchange.com%2fquestions%2f3031564%2ftricky-analysis-proof-involving-the-sum-of-integrals%23new-answer', 'question_page');
    }
    );

    Post as a guest















    Required, but never shown

























    1 Answer
    1






    active

    oldest

    votes








    1 Answer
    1






    active

    oldest

    votes









    active

    oldest

    votes






    active

    oldest

    votes









    1












    $begingroup$

    This is just a generalization of integrability of $f$ on $[a,c]$ and $[c,b]$ implies integrability on $[a,b]$ and
    $$int_a^b f(x) , dx = int_a^c f(x) , dx + int_c^b f(x) , dx $$



    There exist partitions $P_j$ of $[t_{j-1},t_j]$ such that for $j=1,ldots,m$, upper and lower Darboux sums satisfy



    $$U(P_j,f_j) - L(P_j,f_j) < frac{epsilon}{m}$$



    With the partition $P = bigcup_{j=1}^n P_j$ we have



    $$U(P,f) - L(P,f) = sum_{j=1}^n [ U(P_j,f_j) - L(P_j,f_j)] < epsilon$$



    and $f$ is integrable by the Riemann criterion.



    We must also have



    $$L(P,f) leqslant int_a^b f(x) , dx leqslant U(P,f), \ L(P,f) = sum_{j=1}^n L(P_j,f_j) = sum_{j=1}^n int_{t_{j-1}}^{t_j} f_j(x) , dx leqslant sum_{j=1}^n U(P_j,f_j) = U(P,f)$$



    Thus,



    $$-epsilon < -[U(P,f) - L(P,f)] < int_a^b f(x) , dx - sum_{j=1}^n int_{t_{j-1}}^{t_j} f_j(x) , dx < U(P,f) - L(P,f) < epsilon,$$



    which implies (since $epsilon$ can be chosen arbitrarily close to $0$)



    $$int_a^b f(x) , dx = sum_{j=1}^n int_{t_{j-1}}^{t_j} f_j(x) , dx $$






    share|cite|improve this answer









    $endgroup$


















      1












      $begingroup$

      This is just a generalization of integrability of $f$ on $[a,c]$ and $[c,b]$ implies integrability on $[a,b]$ and
      $$int_a^b f(x) , dx = int_a^c f(x) , dx + int_c^b f(x) , dx $$



      There exist partitions $P_j$ of $[t_{j-1},t_j]$ such that for $j=1,ldots,m$, upper and lower Darboux sums satisfy



      $$U(P_j,f_j) - L(P_j,f_j) < frac{epsilon}{m}$$



      With the partition $P = bigcup_{j=1}^n P_j$ we have



      $$U(P,f) - L(P,f) = sum_{j=1}^n [ U(P_j,f_j) - L(P_j,f_j)] < epsilon$$



      and $f$ is integrable by the Riemann criterion.



      We must also have



      $$L(P,f) leqslant int_a^b f(x) , dx leqslant U(P,f), \ L(P,f) = sum_{j=1}^n L(P_j,f_j) = sum_{j=1}^n int_{t_{j-1}}^{t_j} f_j(x) , dx leqslant sum_{j=1}^n U(P_j,f_j) = U(P,f)$$



      Thus,



      $$-epsilon < -[U(P,f) - L(P,f)] < int_a^b f(x) , dx - sum_{j=1}^n int_{t_{j-1}}^{t_j} f_j(x) , dx < U(P,f) - L(P,f) < epsilon,$$



      which implies (since $epsilon$ can be chosen arbitrarily close to $0$)



      $$int_a^b f(x) , dx = sum_{j=1}^n int_{t_{j-1}}^{t_j} f_j(x) , dx $$






      share|cite|improve this answer









      $endgroup$
















        1












        1








        1





        $begingroup$

        This is just a generalization of integrability of $f$ on $[a,c]$ and $[c,b]$ implies integrability on $[a,b]$ and
        $$int_a^b f(x) , dx = int_a^c f(x) , dx + int_c^b f(x) , dx $$



        There exist partitions $P_j$ of $[t_{j-1},t_j]$ such that for $j=1,ldots,m$, upper and lower Darboux sums satisfy



        $$U(P_j,f_j) - L(P_j,f_j) < frac{epsilon}{m}$$



        With the partition $P = bigcup_{j=1}^n P_j$ we have



        $$U(P,f) - L(P,f) = sum_{j=1}^n [ U(P_j,f_j) - L(P_j,f_j)] < epsilon$$



        and $f$ is integrable by the Riemann criterion.



        We must also have



        $$L(P,f) leqslant int_a^b f(x) , dx leqslant U(P,f), \ L(P,f) = sum_{j=1}^n L(P_j,f_j) = sum_{j=1}^n int_{t_{j-1}}^{t_j} f_j(x) , dx leqslant sum_{j=1}^n U(P_j,f_j) = U(P,f)$$



        Thus,



        $$-epsilon < -[U(P,f) - L(P,f)] < int_a^b f(x) , dx - sum_{j=1}^n int_{t_{j-1}}^{t_j} f_j(x) , dx < U(P,f) - L(P,f) < epsilon,$$



        which implies (since $epsilon$ can be chosen arbitrarily close to $0$)



        $$int_a^b f(x) , dx = sum_{j=1}^n int_{t_{j-1}}^{t_j} f_j(x) , dx $$






        share|cite|improve this answer









        $endgroup$



        This is just a generalization of integrability of $f$ on $[a,c]$ and $[c,b]$ implies integrability on $[a,b]$ and
        $$int_a^b f(x) , dx = int_a^c f(x) , dx + int_c^b f(x) , dx $$



        There exist partitions $P_j$ of $[t_{j-1},t_j]$ such that for $j=1,ldots,m$, upper and lower Darboux sums satisfy



        $$U(P_j,f_j) - L(P_j,f_j) < frac{epsilon}{m}$$



        With the partition $P = bigcup_{j=1}^n P_j$ we have



        $$U(P,f) - L(P,f) = sum_{j=1}^n [ U(P_j,f_j) - L(P_j,f_j)] < epsilon$$



        and $f$ is integrable by the Riemann criterion.



        We must also have



        $$L(P,f) leqslant int_a^b f(x) , dx leqslant U(P,f), \ L(P,f) = sum_{j=1}^n L(P_j,f_j) = sum_{j=1}^n int_{t_{j-1}}^{t_j} f_j(x) , dx leqslant sum_{j=1}^n U(P_j,f_j) = U(P,f)$$



        Thus,



        $$-epsilon < -[U(P,f) - L(P,f)] < int_a^b f(x) , dx - sum_{j=1}^n int_{t_{j-1}}^{t_j} f_j(x) , dx < U(P,f) - L(P,f) < epsilon,$$



        which implies (since $epsilon$ can be chosen arbitrarily close to $0$)



        $$int_a^b f(x) , dx = sum_{j=1}^n int_{t_{j-1}}^{t_j} f_j(x) , dx $$







        share|cite|improve this answer












        share|cite|improve this answer



        share|cite|improve this answer










        answered Dec 9 '18 at 6:38









        RRLRRL

        52.7k42573




        52.7k42573






























            draft saved

            draft discarded




















































            Thanks for contributing an answer to Mathematics Stack Exchange!


            • Please be sure to answer the question. Provide details and share your research!

            But avoid



            • Asking for help, clarification, or responding to other answers.

            • Making statements based on opinion; back them up with references or personal experience.


            Use MathJax to format equations. MathJax reference.


            To learn more, see our tips on writing great answers.




            draft saved


            draft discarded














            StackExchange.ready(
            function () {
            StackExchange.openid.initPostLogin('.new-post-login', 'https%3a%2f%2fmath.stackexchange.com%2fquestions%2f3031564%2ftricky-analysis-proof-involving-the-sum-of-integrals%23new-answer', 'question_page');
            }
            );

            Post as a guest















            Required, but never shown





















































            Required, but never shown














            Required, but never shown












            Required, but never shown







            Required, but never shown

































            Required, but never shown














            Required, but never shown












            Required, but never shown







            Required, but never shown







            Popular posts from this blog

            Biblatex bibliography style without URLs when DOI exists (in Overleaf with Zotero bibliography)

            ComboBox Display Member on multiple fields

            Is it possible to collect Nectar points via Trainline?